Laplace of a Unit Step Function

Click For Summary
The discussion revolves around finding the Laplace transform of a piecewise function defined as f(t) = 0 for t < 4 and f(t) = (t-3)^3 for t ≥ 4. The user attempts to express the function using the unit step function u(t-4) and considers changing it to u(t-4)^4, which raises questions about the correct application of the Laplace transform. There is confusion regarding the proper form of the Laplace transform and the integration variable change to u = t - 4. The user seeks clarification and a worked example to better understand the process.
sssssssssss
Messages
5
Reaction score
0

Homework Statement


f(t) = {0, if t<4 and (t-3)^3 if t\geq4


The Attempt at a Solution


I feel like its pretty basic but i can't get it down
I have u(t-4)(t-4)^3

Can i change it to u(t-4)^4?
Then do i multiply it out and take the laplace?
If someone can work it out for me that would be awesome, I have about 20 more and I just need an example, i appologize if that's against the rules.

I want to say the answer is (6/s^4 - 24/s^3 + 48/s^2 - 64/s)e^-4s but that's wrong.
 
Physics news on Phys.org
I don't know if this is the 'laplace' way of doing things, but change the variable of integration to u=t-4. Then what does the integral that defines the laplace transform look like in terms of u?
 
Question: A clock's minute hand has length 4 and its hour hand has length 3. What is the distance between the tips at the moment when it is increasing most rapidly?(Putnam Exam Question) Answer: Making assumption that both the hands moves at constant angular velocities, the answer is ## \sqrt{7} .## But don't you think this assumption is somewhat doubtful and wrong?

Similar threads

  • · Replies 8 ·
Replies
8
Views
2K
  • · Replies 4 ·
Replies
4
Views
3K
  • · Replies 8 ·
Replies
8
Views
2K
Replies
8
Views
1K
Replies
12
Views
2K
  • · Replies 9 ·
Replies
9
Views
3K
  • · Replies 2 ·
Replies
2
Views
2K
  • · Replies 6 ·
Replies
6
Views
5K
Replies
9
Views
2K
  • · Replies 8 ·
Replies
8
Views
2K